$ lim_ntoinftyu_n(x,y)=u(x,y) $ is true for a sequence of harmonic functions satisfying $ lim_ntoinftyint_B |u_n(x, y)-u(x,y)|dxdy=0 .$Proving an elementary integral inequality (in early Dirichlet space material)For the sequence $u_n$, $u_n to +infty iff frac1u_n to 0$square of a harmonic function boundUpper bound for coefficients of a power seriesA version of Casorati-Weierstrass for harmonic functions?Let $u$ harmonic. Then $int_mathbb R^d|u|^2<infty implies u=0$Evaluate $iint_D2x-2y dxdy$ using polar coordinates.Proving the Harnack Inequality for Harmonic FunctionsAlgebraically Decaying Function with a Convergent IntegralProve that $lim_n to infty int_[0,1]x^n, dx = 0$. Where $int$ represents Lebesgue integration.

What's the difference between 'rename' and 'mv'?

How can I prevent hyper evolved versions of regular creatures from wiping out their cousins?

What's the point of deactivating Num Lock on login screens?

Can I ask the recruiters in my resume to put the reason why I am rejected?

If human space travel is limited by the G force vulnerability, is there a way to counter G forces?

Has there ever been an airliner design involving reducing generator load by installing solar panels?

Fully-Firstable Anagram Sets

Is there a hemisphere-neutral way of specifying a season?

How can I tell someone that I want to be his or her friend?

Will google still index a page if I use a $_SESSION variable?

What reasons are there for a Capitalist to oppose a 100% inheritance tax?

Intersection of two sorted vectors in C++

How do conventional missiles fly?

Assassin's bullet with mercury

What is the word for reserving something for yourself before others do?

Is it legal for company to use my work email to pretend I still work there?

I Accidentally Deleted a Stock Terminal Theme

Is it possible to create light that imparts a greater proportion of its energy as momentum rather than heat?

In Romance of the Three Kingdoms why do people still use bamboo sticks when papers are already invented?

What killed these X2 caps?

Blender 2.8 I can't see vertices, edges or faces in edit mode

Is it possible to run Internet Explorer on OS X El Capitan?

How do I write bicross product symbols in latex?

Arrow those variables!



$ lim_ntoinftyu_n(x,y)=u(x,y) $ is true for a sequence of harmonic functions satisfying $ lim_ntoinftyint_B |u_n(x, y)-u(x,y)|dxdy=0 .$


Proving an elementary integral inequality (in early Dirichlet space material)For the sequence $u_n$, $u_n to +infty iff frac1u_n to 0$square of a harmonic function boundUpper bound for coefficients of a power seriesA version of Casorati-Weierstrass for harmonic functions?Let $u$ harmonic. Then $int_mathbb R^d|u|^2<infty implies u=0$Evaluate $iint_D2x-2y dxdy$ using polar coordinates.Proving the Harnack Inequality for Harmonic FunctionsAlgebraically Decaying Function with a Convergent IntegralProve that $lim_n to infty int_[0,1]x^n, dx = 0$. Where $int$ represents Lebesgue integration.













0












$begingroup$



(i)Let $ B= (x, y)inmathbb R^2:x^2+y^2<1 $, and let $ u(x, y) $ be a harmonic function defined on some open set $ U $ containing the closure of $ B $. Prove that
$$ u(0,0)=frac 1piint_Bu(x,y)dxdy .$$
(ii)Suppose, in addition to the above assumption, that $ u_n(x,y)_n=1^infty $ is a sequence of harmonic functions on $ U $ such that
$$ lim_ntoinftyint_B |u_n(x, y)-u(x,y)|dxdy=0 .$$
Show that $ lim_ntoinftyu_n(x,y)=u(x,y) $ for all $ (x,y)in B $.





My attempt:



(i) is rather straightforward:
beginalign
frac 1piint_Bu(x,y)dxdy&=frac 1 piint_B u(r,theta)rdrdtheta\
&=frac 1 piint_0^1 rleft( int_0^2piu(r, theta)dtheta right)dr\
&=frac 1 piint_0^1 r2pi u(0,0)dr\
&=2u(0,0)int_0^1 rdr\
&=u(0,0)
endalign

where we have used the mean value property of harmonic function over a unit ball.



For (ii)
beginalign
&&lim_ntoinftyint_B left|u_n(x,y)-u(x,y)right|dxdy&=0\
&implies&lim_ntoinftyleft|int_B left[u_n(x,y)-u(x,y)right]dxdyright|&=0\
&implies&lim_ntoinftyint_Bleft[ u_n(x,y)-u(x,y)right]dxdy&=0\
&implies&lim_ntoinftyint_Bu_n(x,y)dxdy&=int_Bu(x,y)dxdy\
&implies&pilim_ntoinftyu_n(0,0)&=pi u(0,0)\
&implies&lim_ntoinftyu_n(0,0)&= u(0,0)
endalign

So we have proved $ lim_ntoinftyu_n(x,y)=u(x,y) $ for $ (0,0) $. Can we apply the above technique to an arbitray ball centered at a point in $ B $ to conclude that $ lim_ntoinftyu_n(x,y)=u(x,y) $ is true for all $ (x,y)in B $ ?










share|cite|improve this question









$endgroup$







  • 2




    $begingroup$
    Yes. If $(a,b)$ is point in $B$, then the function $v(x,y) =u(a+x,b+y)$ is also harmonic. Therefore the results above hold for it as well.
    $endgroup$
    – Fnacool
    Mar 29 at 3:03















0












$begingroup$



(i)Let $ B= (x, y)inmathbb R^2:x^2+y^2<1 $, and let $ u(x, y) $ be a harmonic function defined on some open set $ U $ containing the closure of $ B $. Prove that
$$ u(0,0)=frac 1piint_Bu(x,y)dxdy .$$
(ii)Suppose, in addition to the above assumption, that $ u_n(x,y)_n=1^infty $ is a sequence of harmonic functions on $ U $ such that
$$ lim_ntoinftyint_B |u_n(x, y)-u(x,y)|dxdy=0 .$$
Show that $ lim_ntoinftyu_n(x,y)=u(x,y) $ for all $ (x,y)in B $.





My attempt:



(i) is rather straightforward:
beginalign
frac 1piint_Bu(x,y)dxdy&=frac 1 piint_B u(r,theta)rdrdtheta\
&=frac 1 piint_0^1 rleft( int_0^2piu(r, theta)dtheta right)dr\
&=frac 1 piint_0^1 r2pi u(0,0)dr\
&=2u(0,0)int_0^1 rdr\
&=u(0,0)
endalign

where we have used the mean value property of harmonic function over a unit ball.



For (ii)
beginalign
&&lim_ntoinftyint_B left|u_n(x,y)-u(x,y)right|dxdy&=0\
&implies&lim_ntoinftyleft|int_B left[u_n(x,y)-u(x,y)right]dxdyright|&=0\
&implies&lim_ntoinftyint_Bleft[ u_n(x,y)-u(x,y)right]dxdy&=0\
&implies&lim_ntoinftyint_Bu_n(x,y)dxdy&=int_Bu(x,y)dxdy\
&implies&pilim_ntoinftyu_n(0,0)&=pi u(0,0)\
&implies&lim_ntoinftyu_n(0,0)&= u(0,0)
endalign

So we have proved $ lim_ntoinftyu_n(x,y)=u(x,y) $ for $ (0,0) $. Can we apply the above technique to an arbitray ball centered at a point in $ B $ to conclude that $ lim_ntoinftyu_n(x,y)=u(x,y) $ is true for all $ (x,y)in B $ ?










share|cite|improve this question









$endgroup$







  • 2




    $begingroup$
    Yes. If $(a,b)$ is point in $B$, then the function $v(x,y) =u(a+x,b+y)$ is also harmonic. Therefore the results above hold for it as well.
    $endgroup$
    – Fnacool
    Mar 29 at 3:03













0












0








0





$begingroup$



(i)Let $ B= (x, y)inmathbb R^2:x^2+y^2<1 $, and let $ u(x, y) $ be a harmonic function defined on some open set $ U $ containing the closure of $ B $. Prove that
$$ u(0,0)=frac 1piint_Bu(x,y)dxdy .$$
(ii)Suppose, in addition to the above assumption, that $ u_n(x,y)_n=1^infty $ is a sequence of harmonic functions on $ U $ such that
$$ lim_ntoinftyint_B |u_n(x, y)-u(x,y)|dxdy=0 .$$
Show that $ lim_ntoinftyu_n(x,y)=u(x,y) $ for all $ (x,y)in B $.





My attempt:



(i) is rather straightforward:
beginalign
frac 1piint_Bu(x,y)dxdy&=frac 1 piint_B u(r,theta)rdrdtheta\
&=frac 1 piint_0^1 rleft( int_0^2piu(r, theta)dtheta right)dr\
&=frac 1 piint_0^1 r2pi u(0,0)dr\
&=2u(0,0)int_0^1 rdr\
&=u(0,0)
endalign

where we have used the mean value property of harmonic function over a unit ball.



For (ii)
beginalign
&&lim_ntoinftyint_B left|u_n(x,y)-u(x,y)right|dxdy&=0\
&implies&lim_ntoinftyleft|int_B left[u_n(x,y)-u(x,y)right]dxdyright|&=0\
&implies&lim_ntoinftyint_Bleft[ u_n(x,y)-u(x,y)right]dxdy&=0\
&implies&lim_ntoinftyint_Bu_n(x,y)dxdy&=int_Bu(x,y)dxdy\
&implies&pilim_ntoinftyu_n(0,0)&=pi u(0,0)\
&implies&lim_ntoinftyu_n(0,0)&= u(0,0)
endalign

So we have proved $ lim_ntoinftyu_n(x,y)=u(x,y) $ for $ (0,0) $. Can we apply the above technique to an arbitray ball centered at a point in $ B $ to conclude that $ lim_ntoinftyu_n(x,y)=u(x,y) $ is true for all $ (x,y)in B $ ?










share|cite|improve this question









$endgroup$





(i)Let $ B= (x, y)inmathbb R^2:x^2+y^2<1 $, and let $ u(x, y) $ be a harmonic function defined on some open set $ U $ containing the closure of $ B $. Prove that
$$ u(0,0)=frac 1piint_Bu(x,y)dxdy .$$
(ii)Suppose, in addition to the above assumption, that $ u_n(x,y)_n=1^infty $ is a sequence of harmonic functions on $ U $ such that
$$ lim_ntoinftyint_B |u_n(x, y)-u(x,y)|dxdy=0 .$$
Show that $ lim_ntoinftyu_n(x,y)=u(x,y) $ for all $ (x,y)in B $.





My attempt:



(i) is rather straightforward:
beginalign
frac 1piint_Bu(x,y)dxdy&=frac 1 piint_B u(r,theta)rdrdtheta\
&=frac 1 piint_0^1 rleft( int_0^2piu(r, theta)dtheta right)dr\
&=frac 1 piint_0^1 r2pi u(0,0)dr\
&=2u(0,0)int_0^1 rdr\
&=u(0,0)
endalign

where we have used the mean value property of harmonic function over a unit ball.



For (ii)
beginalign
&&lim_ntoinftyint_B left|u_n(x,y)-u(x,y)right|dxdy&=0\
&implies&lim_ntoinftyleft|int_B left[u_n(x,y)-u(x,y)right]dxdyright|&=0\
&implies&lim_ntoinftyint_Bleft[ u_n(x,y)-u(x,y)right]dxdy&=0\
&implies&lim_ntoinftyint_Bu_n(x,y)dxdy&=int_Bu(x,y)dxdy\
&implies&pilim_ntoinftyu_n(0,0)&=pi u(0,0)\
&implies&lim_ntoinftyu_n(0,0)&= u(0,0)
endalign

So we have proved $ lim_ntoinftyu_n(x,y)=u(x,y) $ for $ (0,0) $. Can we apply the above technique to an arbitray ball centered at a point in $ B $ to conclude that $ lim_ntoinftyu_n(x,y)=u(x,y) $ is true for all $ (x,y)in B $ ?







complex-analysis proof-verification harmonic-functions






share|cite|improve this question













share|cite|improve this question











share|cite|improve this question




share|cite|improve this question










asked Mar 29 at 2:57









user549397user549397

1,6591418




1,6591418







  • 2




    $begingroup$
    Yes. If $(a,b)$ is point in $B$, then the function $v(x,y) =u(a+x,b+y)$ is also harmonic. Therefore the results above hold for it as well.
    $endgroup$
    – Fnacool
    Mar 29 at 3:03












  • 2




    $begingroup$
    Yes. If $(a,b)$ is point in $B$, then the function $v(x,y) =u(a+x,b+y)$ is also harmonic. Therefore the results above hold for it as well.
    $endgroup$
    – Fnacool
    Mar 29 at 3:03







2




2




$begingroup$
Yes. If $(a,b)$ is point in $B$, then the function $v(x,y) =u(a+x,b+y)$ is also harmonic. Therefore the results above hold for it as well.
$endgroup$
– Fnacool
Mar 29 at 3:03




$begingroup$
Yes. If $(a,b)$ is point in $B$, then the function $v(x,y) =u(a+x,b+y)$ is also harmonic. Therefore the results above hold for it as well.
$endgroup$
– Fnacool
Mar 29 at 3:03










0






active

oldest

votes












Your Answer





StackExchange.ifUsing("editor", function ()
return StackExchange.using("mathjaxEditing", function ()
StackExchange.MarkdownEditor.creationCallbacks.add(function (editor, postfix)
StackExchange.mathjaxEditing.prepareWmdForMathJax(editor, postfix, [["$", "$"], ["\\(","\\)"]]);
);
);
, "mathjax-editing");

StackExchange.ready(function()
var channelOptions =
tags: "".split(" "),
id: "69"
;
initTagRenderer("".split(" "), "".split(" "), channelOptions);

StackExchange.using("externalEditor", function()
// Have to fire editor after snippets, if snippets enabled
if (StackExchange.settings.snippets.snippetsEnabled)
StackExchange.using("snippets", function()
createEditor();
);

else
createEditor();

);

function createEditor()
StackExchange.prepareEditor(
heartbeatType: 'answer',
autoActivateHeartbeat: false,
convertImagesToLinks: true,
noModals: true,
showLowRepImageUploadWarning: true,
reputationToPostImages: 10,
bindNavPrevention: true,
postfix: "",
imageUploader:
brandingHtml: "Powered by u003ca class="icon-imgur-white" href="https://imgur.com/"u003eu003c/au003e",
contentPolicyHtml: "User contributions licensed under u003ca href="https://creativecommons.org/licenses/by-sa/3.0/"u003ecc by-sa 3.0 with attribution requiredu003c/au003e u003ca href="https://stackoverflow.com/legal/content-policy"u003e(content policy)u003c/au003e",
allowUrls: true
,
noCode: true, onDemand: true,
discardSelector: ".discard-answer"
,immediatelyShowMarkdownHelp:true
);



);













draft saved

draft discarded


















StackExchange.ready(
function ()
StackExchange.openid.initPostLogin('.new-post-login', 'https%3a%2f%2fmath.stackexchange.com%2fquestions%2f3166685%2flim-n-to-inftyu-nx-y-ux-y-is-true-for-a-sequence-of-harmonic-function%23new-answer', 'question_page');

);

Post as a guest















Required, but never shown

























0






active

oldest

votes








0






active

oldest

votes









active

oldest

votes






active

oldest

votes















draft saved

draft discarded
















































Thanks for contributing an answer to Mathematics Stack Exchange!


  • Please be sure to answer the question. Provide details and share your research!

But avoid


  • Asking for help, clarification, or responding to other answers.

  • Making statements based on opinion; back them up with references or personal experience.

Use MathJax to format equations. MathJax reference.


To learn more, see our tips on writing great answers.




draft saved


draft discarded














StackExchange.ready(
function ()
StackExchange.openid.initPostLogin('.new-post-login', 'https%3a%2f%2fmath.stackexchange.com%2fquestions%2f3166685%2flim-n-to-inftyu-nx-y-ux-y-is-true-for-a-sequence-of-harmonic-function%23new-answer', 'question_page');

);

Post as a guest















Required, but never shown





















































Required, but never shown














Required, but never shown












Required, but never shown







Required, but never shown

































Required, but never shown














Required, but never shown












Required, but never shown







Required, but never shown







Popular posts from this blog

Triangular numbers and gcdProving sum of a set is $0 pmod n$ if $n$ is odd, or $fracn2 pmod n$ if $n$ is even?Is greatest common divisor of two numbers really their smallest linear combination?GCD, LCM RelationshipProve a set of nonnegative integers with greatest common divisor 1 and closed under addition has all but finite many nonnegative integers.all pairs of a and b in an equation containing gcdTriangular Numbers Modulo $k$ - Hit All Values?Understanding the Existence and Uniqueness of the GCDGCD and LCM with logical symbolsThe greatest common divisor of two positive integers less than 100 is equal to 3. Their least common multiple is twelve times one of the integers.Suppose that for all integers $x$, $x|a$ and $x|b$ if and only if $x|c$. Then $c = gcd(a,b)$Which is the gcd of 2 numbers which are multiplied and the result is 600000?

Ingelân Ynhâld Etymology | Geografy | Skiednis | Polityk en bestjoer | Ekonomy | Demografy | Kultuer | Klimaat | Sjoch ek | Keppelings om utens | Boarnen, noaten en referinsjes Navigaasjemenuwww.gov.ukOffisjele webside fan it regear fan it Feriene KeninkrykOffisjele webside fan it Britske FerkearsburoNederlânsktalige ynformaasje fan it Britske FerkearsburoOffisjele webside fan English Heritage, de organisaasje dy't him ynset foar it behâld fan it Ingelske kultuergoedYnwennertallen fan alle Britske stêden út 'e folkstelling fan 2011Notes en References, op dizze sideEngland

Հադիս Բովանդակություն Անվանում և նշանակություն | Դասակարգում | Աղբյուրներ | Նավարկման ցանկ